Chapter 23: Nursing Management: Integumentary Problems

Lakukan tugas rumah & ujian kamu dengan baik sekarang menggunakan Quizwiz!

A patient comes to the clinic and informs the nurse that the patient thinks that he or she may have been bitten or stung about three or four weeks ago. A ringlike rash is now present on the area and the patient states he or she has been feeling "flu-like". What question pertaining to this condition should the nurse ask the patient? A. "Have you removed a tick?" B. "Were you stung by a wasp?" C. "Have you been itching all over? D. "Have you noticed any bugs under your mattress?"

A. "Have you removed a tick?" Rationale A tick bite may cause Lyme disease, which often presents with a ringlike rash that is warm, itchy, and painful three to four weeks after the bite, along with flu-like symptoms. Wasps sting rather than bite; a sting causes intense, burning local pain accompanied by swelling and itching. A bedbug bite causes a wheal surrounded by a vivid flare. Body lice bites result in minute red, noninflamed points flush with the skin that progress to popular, wheal-like lesions. p. 416

A nurse advises a patient with skin that is extremely sensitive to the sun to select a sunscreen which blocks both UVA and UVB rays. Which statements by the patient indicate that the teaching is effective? Select all that apply. A. "The sunscreen should be water resistant." B. "The sunscreen should have an SPF of at least 15." C. "The sunscreen should be labeled as broad spectrum." D. "The sunscreen should have benzophenone as the major constituent." E. "The sunscreen should have para-aminobenzoic acid (PABA) as the major constituent."

A. "The sunscreen should be water resistant." C. "The sunscreen should be labeled as broad spectrum." D. "The sunscreen should have benzophenone as the major constituent." Rationale Some sunscreens are water resistant and should be used during swimming. Only a broad spectrum sunscreen blocks both UVA and UVB rays. Therefore the patient must purchase a sunscreen which is labeled as "broad spectrum." Benzophenones are substances that block both UVA and UVB rays, so the patient should check if the sunscreen contains them. A sunscreen with SPF 15 is not sufficient to protect sensitive skin. Sunscreens having SPF 30 are recommended for such patients. PABA is useful for blocking UVB rays but is no longer used in sunscreens, because it is known to cause contact dermatitis and stains clothing. pp. 407-408

Which patient would be more likely to have the highest risk of developing malignant melanoma? A. A fair-skinned woman who uses a tanning booth regularly B. An African American patient with a family history of cancer C. An adult who required phototherapy as an infant for the treatment of hyperbilirubinemia D. A Hispanic male with a history of psoriasis and eczema who responded poorly to treatment

A. A fair-skinned woman who uses a tanning booth regularly Rationale Risk factors for malignant melanoma include a fair complexion and exposure to ultraviolet light. Psoriasis, eczema, short-duration phototherapy, and a family history of other cancers are less likely to be linked to malignant melanoma. pp. 410-411

A nurse works in a dermatology clinic and counsels many patients in a day. Which patients should the nurse consider at high risk of developing skin malignancies? Select all that apply. A. A farmer B. A school teacher C. A baseball player D. A software engineer E. A construction worker

A. A farmer C. A baseball player E. A construction worker Rationale Sun exposure is the main risk factor for development of skin malignancies. People in occupations that involve outdoor activities are more prone to develop skin malignancies. A construction worker, a farmer, and a baseball player are exposed to sunlight throughout the day. Therefore these professionals may have high risk of skin malignancies. A software engineer and a school teacher are involved in indoor activities and are not exposed to sunlight. Therefore they have less risk of developing skin malignancies. pp. 407-409

The nurse is caring for patients who have undergone various cosmetic topical procedures and are experiencing side effects. Which patient had the epidermis and top dermal layer removed by application of aluminum oxide? A. A patient with a light pink tone skin for about 24 hours B. A patient with moderate swelling and crusting for a week C. A patient with photosensitivity with slight irritation of the skin D. A patient with erythema, swelling, flaking, and changes in pigmentation

A. A patient with a light pink tone skin for about 24 hours Rationale Microdermabrasion is the removal of the epidermis and top dermal layer by the application of aluminum oxide. A light pink tone skin that resolves within 24 hours is a side effect of microdermabrasion. A patient with moderate swelling and crusting that lasts for about a week may have undergone treatment with chemical peels. A photosensitive patient with slight skin irritation may have undergone treatment with α-hydroxy acids at low concentrations. A patient with erythema, swelling, flaking, and pigmentation changes may have undergone treatment with tretinoin. p. 426

The nurse assesses an elevated, dry, hyperkeratotic, scaly papule in an older adult patient. What condition does the assessment data correlate? A. Actinic keratosis B. Dysplastic nevus C. Basal cell carcinoma D. Squamous cell carcinoma

A. Actinic keratosis Rationale Actinic keratosis manifests clinically as dry, scaly, hyperkeratotic papules, either flat or elevated. A dysplastic or atypical nevus is often larger than 5 mm. It features irregular borders, which may be notched; variegated color (e.g., tan, brown, black, red, or pink) within a single mole; and at least one flat portion, often at the edge of the mole. Basal cell carcinoma is characterized by a small, slowly enlarging papule with semitranslucent or pearly borders. Squamous cell carcinoma appears as a thin, scaly, erythematous plaque that does not invade the dermis. STUDY TIP: Remember that intelligence plays a vital role in your ability to learn. However, being smart involves more than just intelligence. Being practical and applying common sense are also parts of the learning experience. p. 411

A nurse is assessing a patient with a bee sting. To relieve pain, what will most likely be included in the patient's treatment plan? Select all that apply. A. Antihistamines B. Antipruritic lotion C. Cold compresses D. Chlorocyclohexane E. Gamma-Benzene hexachloride

A. Antihistamines B. Antipruritic lotion C. Cold compresses Rationale The symptoms of a bee sting are burning pain, swelling, and itching in the affected area. Cool compresses are used to reduce the burning sensation. Antipruritic lotion is used to reduce itching. Antihistamines are used for allergies. Chlorocyclohexane is used to control bedbugs. Gamma-benzene hexachloride is used to treat pediculosis. p. 416

A patient with a skin infection is prescribed a topical ointment. What action should the nurse take when applying this treatment? A. Applying a thin layer of the ointment on the affected area B. Spreading the ointment on the affected area in a circular motion C. Applying a thick layer of ointment with a gloved hand for better results D. Applying the ointment on the affected area and then applying a dressing

A. Applying a thin layer of the ointment on the affected area Rationale A thin layer of the ointment should be applied to the skin. Thickly applied ointment wastes the medication and leaves the skin greasy. The ointment should be spread evenly in a downward motion on the affected area. The ointment should be applied on the dressing, which is then placed on the affected area. A thin layer of the ointment should be applied with a gloved hand. p. 421

Which nursing intervention would be most helpful in managing a patient newly admitted with cellulitis of the right foot? A. Applying warm, moist heat B. Encouraging frequent ambulation C. Not elevating the affected extremity D. Wrapping the foot snugly in blankets

A. Applying warm, moist heat Rationale The application of warm, moist heat speeds the resolution of inflammation and infection when accompanied by appropriate antibiotic therapy. It does this by increasing local circulation to the affected area to bring macrophages to the area and carry off cellular debris. Immobilization and elevation are also used. Snug blankets would not be helpful and could decrease circulation to this sensitive tissue. Elevation and immobilization, not frequent ambulation and nonelevation of the extremity, would be beneficial for healing. Test-Taking Tip: Read the question carefully before looking at the answers: (1) Determine what the question is really asking; look for key words; (2) Read each answer thoroughly and see if it completely covers the material asked by the question; (3) Narrow the choices by immediately eliminating answers you know are incorrect. p. 414

A patient has been prescribed isotretinoin for severe acne. What assessment should the nurse perform before administering the drug? Select all that apply. A. Assess liver function test. B. Assess the patient's fluid intake. C. Perform pulmonary function tests. D. Ask the patient whether she is pregnant. E. Ask the patient whether she is planning to conceive.

A. Assess liver function test. D. Ask the patient whether she is pregnant. E. Ask the patient whether she is planning to conceive. Rationale The drug isotretinoin is known to interfere with liver functions and may also have teratogenic effects. Therefore liver function tests should be conducted to test for abnormalities. The nurse should assess whether the patient is pregnant or planning to become pregnant before administering the drug. There is no need for pulmonary function tests. The amount of fluid intake by the patient does not contribute to the patient's condition as a result of taking this medication. p. 417

A nurse is assessing a woman who has nodulocystic acne. While administering isotretinoin, which interventions are necessary to ensure patient safety? Select all that apply. A. Conduct a pregnancy test. B. Conduct a liver function test. C. Conduct a cholesterol level test. D. Perform a hemoglobin level test. E. Determine if the patient has a history of depression.

A. Conduct a pregnancy test. B. Conduct a liver function test. C. Conduct a cholesterol level test. E. Determine if the patient has a history of depression. Rationale Isotretinoin is used for the treatment of nodulocystic acne. A pregnancy test should be done before administering isotretinoin, because the drug is contraindicated in pregnancy. Isotretinoin can cause serious damage to the fetus. A liver function test is used to check abnormalities in liver function. The cholesterol level is checked because this drug increases blood cholesterol. A liver function test is used to check for abnormalities in liver function. If the patient has a history of depression, use of isotretinoin is prohibited. Hemoglobin levels are unrelated to administration of isotretinoin. p. 417

A patient has a growth on the bottom of one foot, which is growing inward, is painful when pressure is applied, and has interrupted skin markings. Which treatments may the patient receive? Select all that apply. A. Cryosurgery B. White petroleum C. Silver sulfadiazine D. Topical immunotherapy E. Blunt dissection with scissors

A. Cryosurgery D. Topical immunotherapy Rationale The patient has a plantar wart with interrupted markings that has grown inward because of the pressure exerted by walking or standing; it is painful when pressure is applied. Treatment for plantar warts consists of topical immunotherapy, cryosurgery, and the use of salicylic acid. White petroleum is applied as a symptomatic treatment to lesions caused by herpes simplex virus. Silver sulfadiazine is applied as a symptomatic treatment to ruptured vesicles caused by herpes zoster. Blunt dissection with scissors or curette is performed as a treatment for verruca vulgaris. p. 415

In a patient admitted with cellulitis of the left foot, which clinical manifestation would the nurse expect to find on assessment of the left foot? A. Erythema and swelling B. Pallor and poor turgor C. Cyanosis and coolness D. Edema and brown skin discoloration

A. Erythema and swelling Rationale Cellulitis is a diffuse, acute inflammation of the skin. It is characterized by erythematous, swelling, heat, and tenderness in the affected area. These changes accompany the processes of inflammation and infection. p. 414

A patient diagnosed with malignant melanoma has been prescribed vemurafenib. The nurse recalls that the medication was prescribed on the basis of what test? A. Genetic testing B. Clark level testing C. Microdermabrasion D. Breslow's measurement

A. Genetic testing Rationale Vemurafenib is prescribed only to those malignant melanoma patients whose tumors express a specific gene mutation. This genetic mutation is determined using genetic testing. Microdermabrasion is not a test, but a common cosmetic procedure done to improve the appearance of photo-damaged and wrinkled skin. Clark's level and Breslow's measurement are used to determine the depth and thickness of the tumor, which helps to assess the prognosis of melanoma. p. 413

The nurse receives a patient from the postanesthesia care unit (PACU) who has just undergone a rhytidectomy to remove the redundant soft tissue of the neck as a result of sun damage. Which postoperative nursing interventions are appropriate? Select all that apply. A. Ice packs B. Mild analgesics C. Circulation assessments D. Loose, noncompressing dressings E. Supporting the patient's decision for elective surgery

A. Ice packs B. Mild analgesics C. Circulation assessments E. Supporting the patient's decision for elective surgery Rationale Application of ice for the first 24 to 48 hours assists in preventing hematoma formation and reduces swelling. Usually, mild analgesics are sufficient to keep the patient comfortable. If the surgery involves an alteration in the skin's circulation, the nurse should monitor the patient for adequate circulation. Regardless of the patient's reasons for seeking elective surgery, the nurse must maintain a supportive, nonjudgmental attitude. Supportive, compressive dressings rather than loose dressings may be necessary early in the postoperative period. pp. 425-426

The patient is in the hospital for a surgical procedure and has dry skin and pruritus on the legs that cause the patient to scratch at the skin uncontrollably. What measures can the nurse use to help stop the itch/scratch cycle? Select all that apply. A. Moisturize the skin on the legs. B. Provide a warm blanket and room. C. Administer antihistamines at bedtime. D. Use careful hand washing after rubbing her legs. E. Cleanse the legs with a saline solution twice daily.

A. Moisturize the skin on the legs. C. Administer antihistamines at bedtime. Rationale Moisturizing the skin to decrease the dryness and the itch sensation and bedtime antihistamines to decrease a potential allergic reaction and provide some sedation will help the patient sleep, because pruritus is often worse at night and the patient needs sleep for healing. Using nonallergic sheets also may help. Anything causing vasodilation, such as warmth or rubbing, should be avoided. Saline solution would only further dry the skin and so would not be used on the patient's legs. p. 424

The patient has been diagnosed with tinea unguium (onychomycosis) under the nails and does not like the oral antifungal medication. What is the best alternate treatment the nurse should describe for the patient? A. Nail avulsion B. Antifungal cream C. Thinning of fingernails D. Soaking nails in salt water

A. Nail avulsion Rationale Nail avulsion is the best alternate treatment to the oral antifungal medication. Antifungal cream is minimally effective. Thinning fingernails is not needed if the tinea unguium is under the toenails. Soaking the nails will not be helpful. p. 418

The patient with a stage IV pressure ulcer on the coccyx will need a skin graft to close the wound. What postoperative care should the nurse expect to use to facilitate healing? A. No straining of the grafted site B. The wound will be exposed to air C. Soft tissue expansion will be done daily D. The pressure dressing will not be removed

A. No straining of the grafted site Rationale Straining or stretching of the grafted site must be avoided to allow the graft to be vascularized and fixed to the new site for healing. The wound may or may not be exposed to air depending on the type of graft, and the donor site will be covered with a protective dressing to prevent further damage. Soft tissue expansion and pressure dressings will not be used after this wound's skin graft. pp. 426-427

The nurse is caring for four patients with skin infections. In which patient does the nurse suspect infection with group A β-hemolytic streptococci? A. Patient 1: hot red, hard, painfukl plaques with distinct borders. B. Patient 2: Hot, red, and swollen area that has no clear border and is sore to the touch. C. Patient 3: Linear distribution of vesciles on the right side of the back only. D. Patient 4: Reddish lesion, sore to the touch, around a hair follice.

A. Patient 1: hot red, hard, painfukl plaques with distinct borders. Rationale Patient 1, with hot, red, painful, well-demarcated indurated plaques, may have erysipelas; it is an infection caused by group A β-hemolytic streptococci. Patient 2, with a hot, tender, erythematous, and edematous areas with diffuse border, may have cellulitis, which is caused by Staphylococcus and Streptococcus. Patient 3, with a unilateral linear distribution of vesicles, may have herpes zoster (shingles), a viral infection. Patient 4, with a tender, erythematous area around a hair follicle, may have a furuncle, which is caused by staphylococci. p. 414

A patient who is undergoing therapy for a dermatologic condition is experiencing persistent itching and burns on the skin from sun exposure. Which therapy should the nurse suspect the patient has been undergoing? A. Phototherapy B. Radiation therapy C. Laser technology therapy D. Immunomodulator therapy

A. Phototherapy Rationale A patient undergoing phototherapy may take psoralen, a photosensitizing drug. The side effects of psoralen include sunburns and persistent pruritus. The side effects of radiation therapy include permanent hair loss of the irradiated areas, telangiectasia, atrophy, hyperpigmentation, and depigmentation. Laser therapy does not cause any cumulative cellular changes or damage because laser light does not accumulate in the body. Immunomodulator therapy is used to treat atopic dermatitis; the side effects of immunomodulators are minimal and may include burning or a feeling of heat at the area where it is applied. Test-Taking Tip: You have at least a 25 percent chance of selecting the correct response in multiple-choice items. If you are uncertain about a question, eliminate the choices that you believe are wrong and then call on your knowledge, skills, and abilities to choose from the remaining responses. p. 420

A patient exhibits small, red pruritic lesions between the fingers and toes. On examination the nurse discovers that the lesions appear in a pattern of small lines. Which condition do these symptoms most likely represent? A. Scabies B. Varicella C. Contusions D. Herpes zoster

A. Scabies Rationale Scabies are contagious microscopic mites that manifest as linear burrows, frequently seen between the fingers and toes. A scabies infestation causes intense itching, especially at night. Varicella (chickenpox) is a highly contagious airborne disease that appears as an itchy vesicular rash. A contusion is a bruise that results from the leakage of blood from capillary vessels into surrounding tissue. Herpes zoster (shingles) is a viral disease seen in older adults that manifests as a painful rash with blisters. Herpes zoster may occur as a reactivation of the varicella virus, which lies dormant in the ganglion after a primary case of chickenpox. p. 416

The nurse is caring for a patient with purulent drainage from an inflamed site. Which are the most appropriate actions for the nurse to take? Select all that apply. A. Soak the affected area in salt water. B. Use cool, sterile water for wet dressing. C. Use tap water at room temperature for wet dressing. D. Leave the wet dressing in place for 10 to 30 minutes. E. Replace the wet dressing if the patient's skin appears macerated.

A. Soak the affected area in salt water. D. Leave the wet dressing in place for 10 to 30 minutes. Rationale Purulent drainage indicates infection or inflammation and the affected area should be soaked in salt water or a prescribed solution. Any wet dressings should be left on the affected area for 10 to 30 minutes. If a drying effect is needed, then the wet dressing should be left on the affected area for 20 minutes. A wet dressing need not be sterile unless specified. Normal tap water can be used for a wet dressing, but cool water must be used because of inflammation. If the patient's skin appears macerated, the wet dressing should be discontinued for two to three hours. Test-Taking Tip: Purulent drainage indicates oozing of pus from the affected area. Select the appropriate interventions that may help alleviate the purulent drainage and inflammation in the patient. pp. 422-423

A patient with psoriasis tells the nurse that the patient has quit his or her job as a restaurant hostess because the patient believes the lesions on his or her hands and arms are unattractive to customers. Which nursing diagnosis best describes this patient response? A. Social isolation related to fear of rejection B. Ineffective coping related to lack of social support C. Impaired skin integrity related to presence of lesions D. Ineffective health maintenance because of presence of lesions

A. Social isolation related to fear of rejection Rationale The chronicity of psoriasis can be severe and disabling as people withdraw from social contacts because of visible lesions. Quality of life is also affected negatively. The information presented does not indicate the patient does not have support. Impaired skin integrity is not a priority diagnosis. There is no information to indicate the patient has ineffective health maintenance. pp. 417, 419

Which treatments would be beneficial for a patient with papular, wheal-like lesions and severe pruritus? Select all that apply. A. Spinosad B. Doxycycline C. Chlorocyclohexane D. γ-Benzene hexachloride E. Permethrin topical solution

A. Spinosad D. γ-Benzene hexachloride Rationale Papular, wheal-like lesions and severe pruritus indicate that the patient has been bitten by lice. Spinosad can be used to treat the scalp and hair to rid them of lice. Pyrethrins or γ-benzene hexachloride can also be used to treat lice on various parts of the body. Doxycycline is an antibiotic used in the treatment of tick bites. Chlorocyclohexane is used to control bedbugs. Permethrin topical solution is used in the treatment of scabies. p. 416

A patient has sharply demarcated red plaques on the face that are hot, hard, and painful, along with a body temperature of 101 o F as a result of a β-hemolytic Streptococcus infection. About what treatment should the nurse educate the patient? A. Systemic penicillin B. Warm, moist compresses on the plaques C. Application of retapamulin to the affected area D. Cleansing with antistaphylococcal soap and water

A. Systemic penicillin Rationale The patient has demarcated red plaques that are hot, indurated, and painful, as well as an increased body temperature; this indicates erysipelas. Erysipelas is caused by β-hemolytic streptococci. Systemic antibiotics such as penicillin are used to treat erysipelas; hospitalization is often required. Warm, moist compresses are used for skin infections such as folliculitis, furuncle, and furunculosis. Topical antibiotics such as retapamulin are used to treat such skin infections such as impetigo. Cleansing with antistaphylococcal soap and water is performed to treat folliculitis. Test-Taking Tip: Avoid looking for an answer pattern or code. There may be times when four or five consecutive questions have the same letter or number as the correct answer. p. 414

In teaching a patient who is using topical fluorouracil (5-FU) to treat actinic keratosis, the nurse should tell the patient which of these? Select all that apply. A. Systemic side effects are very rare. B. The patient will look worse before looking better. C. The patient needs to avoid sunlight during treatment. D. Treatment with this medication is limited to one week. E. Abruptly discontinuing the use of the medication may cause a reappearance of the actinic keratosis.

A. Systemic side effects are very rare. B. The patient will look worse before looking better. C. The patient needs to avoid sunlight during treatment. Rationale 5-FU is a topical treatment for actinic keratosis, a premalignant lesion, and certain skin malignancies. Systemic absorption of the drug is minimal and systemic side effects are virtually nonexistent. Because 5-FU is a photosensitizing drug, instruct the patient to avoid sunlight during treatment. Teach patients about the effect of the medication and that they will look worse before they look better. Treatments last from two to six weeks. Abruptly discontinuing 5-FU does not cause the condition to return. STUDY TIP: Identify your problem areas that need attention. Do not waste time on restudying information you know. p. 421

The nurse should teach a patient who is taking which drug to avoid prolonged sun exposure? A. Tetracycline B. Ipratropium C. Morphine sulfate D. Oral contraceptives

A. Tetracycline Rationale Several antibiotics, including tetracycline, may cause photosensitivity. This is not the case with ipratropium, morphine, or oral contraceptives. p. 408

A nurse observes that a group of patients has a darker skin tone after being outside for a period of time. The nurse recalls that the reason for this finding is what? A. The UVA rays of the sun cause tanning. B. The UVB rays of the sun cause sunburn. C. The UVC rays of the sun cause collagen damage. D. The UVC rays of the sun cause increased melanin production.

A. The UVA rays of the sun cause tanning. Rationale Sunlight is composed of ultraviolet rays and visible light rays. Extended exposure to the sun causes the skin to tan. The sun's UVA rays increase the production of melanin in the skin, leading to a tan. UVB rays cause sunburn. UVC rays do not reach the earth because they are blocked by the atmosphere. Both UVA and UVB rays can cause collagen damage and accelerate skin aging. Melanin production increases in response to sunlight exposure. UVC rays alone do not increase melanin production. p. 407

A nurse is assessing a patient who has numerous nevi on the face. The nurse determines that the nevi are not cancerous. Which observations led the nurse to conclude that the nevi are normal? Select all that apply. A. The nevi are dark in color. B. The nevi are well circumscribed. C. The skin over the nevi is eroded. D. There is discharge from the nevi. E. The nevi are larger than 5 mm in size.

A. The nevi are dark in color. B. The nevi are well circumscribed. Rationale Normal nevi are well circumscribed and dark in color. Atypical or dysplastic nevi are larger than 5 mm in size. Eroded skin around the nevi and discharge of pus from the nevi are not associated with normal nevi. They are associated with skin cancer. p. 419

Which patient with a skin condition should be treated with 5% permethrin topical solution? A. The patient with scabies B. The patient with pediculosis C. The patient with tinea corporis D. The patient with Lyme disease

A. The patient with scabies Rationale Scabies is caused by a mite, Sarcoptes scabiei. Scabies is marked by severe body itching, usually at night, and is treated with 5% permethrin solution, an insect repellent. A patient with pediculosis may be bitten by lice. γ-Benzene hexachloride is used to treat lice. Lyme disease is caused by Borrelia burgdorferi, which is transmitted through tick bites. The patient experiences a rash and flu-like symptoms. Oral antibiotics such as doxycycline and tetracycline are effective against Lyme disease. Tinea corporis is caused by various dermatophytes. It is treated with cool compresses and creams of ketoconazole, miconazole, and clotrimazole. Test-Taking Tip: Every insect bite has a characteristic manifestation. Identify the type of insect bite and try to recall the treatment for that bite. p. 416

Which cosmetic procedure improves the appearance of both acne scarring and actinic and seborrheic keratoses? A. Use of chemical peels B. Application of tretinoin C. Use of α-hydroxy acids D. Use of microdermabrasion

A. Use of chemical peels Rationale The use of chemical peels improves the appearance of skin that has been damaged because of acne scarring and actinic and seborrheic keratosis. α-hydroxy acids can smooth photodamaged, wrinkled, or acne-scarred skin. The application of tretinoin can improve the appearance of photodamaged skin and treat fine wrinkling; tretinoin also reduces actinic keratoses. With the use of microdermabrasion, photodamaged, wrinkled, and acne-scarred skin can be smoothed. p. 426

Which cosmetic procedure is teratogenic? A. Use of tretinoin B. Use of chemical peels C. Use of α-hydroxy acids D. Use of microdermabrasion

A. Use of tretinoin Rationale Tretinoin is a teratogenic drug that can cause congenital abnormalities and should not be used during pregnancy. Chemical peels cause redness and photosensitivity. α-hydroxy acids cause photosensitivity, slight irritation, and severe redness. Microdermabrasion causes photosensitivity of the skin. p. 426

The patient has had rashes and alopecia. What vitamin in which foods should be encouraged as a nutritional aid to these problems? A. Vitamin B 6 in liver, cauliflower, salmon, and carrots B. Vitamin C in peppers, dark leafy greens, broccoli, and kiwi C. Vitamin A in sweet potatoes, carrots, and dark leafy greens D. Vitamin D in canned salmon, sardines, fortified dairy, and eggs

A. Vitamin B 6 in liver, cauliflower, salmon, and carrots Rationale A deficiency of Vitamin B 6 may result in rashes and alopecia. Eating foods with biotin will help decrease these problems. Vitamins A and C are needed for wound healing. Vitamin D is needed for bone and body health. p. 409

Which white blood cell count is the best indicator that a 54-year-old patient with cellulitis has recovered from the infection? A. 2000/mm 3 B. 5000/mm 3 C. 13,000/mm 3 D. 16,500/mm 3

B. 5000/mm 3 Rationale The normal white blood cell count is 4000 to 11,000 cells/mm 3, according to most laboratory reference books. For this reason, the patient's level would have been deemed normal if it was 5000/mm 3. A white blood cell count of 2000/mm 3 is categorized as leukopenia and is abnormal. White blood cell counts of 13,000 or 16,500 mm 3 would indicate continued infection. p. 414

The nurse should recognize which patient is likely to have the poorest prognosis? A. A 60-year-old diagnosed with nodular ulcerative basal cell carcinoma B. A 59-year-old man who is being treated for stage IV malignant melanoma C. A 70-year-old woman who has been diagnosed with late squamous cell carcinoma (SCC) D. A 51-year-old woman whose biopsy has revealed superficial squamous cell carcinoma

B. A 59-year-old man who is being treated for stage IV malignant melanoma Rationale Late detection of malignant melanoma is associated with a poor outcome. Basal cell carcinomas often have very effective treatment success rates. Although late SCC has worse outcomes than superficial SCC, these are both exceeded in mortality by late-stage malignant melanoma. p. 411

A patient with severe dermatitis is prescribed a therapeutic bath. What actions should the nurse take when providing the bath to this patient? Select all that apply. A. Have the patient soak in a bath for 20 minutes once a day. B. Add sodium bicarbonate, but not oils, directly to the bath water. C. Fill the bathtub until the affected area is covered with cool water. D. Apply cream to the affected area after getting out of the bathtub. E. Dry the affected area by rubbing with a towel after getting out of the bathtub.

B. Add sodium bicarbonate, but not oils, directly to the bath water. D. Apply cream to the affected area after getting out of the bathtub. Rationale Sodium bicarbonate can be directly added to the bath water of a patient with severe dermatitis to facilitate healing. To sustain the hydrating effect, creams and ointment emollients should be applied to the affected area. Oils are not preferred because the addition of oils makes the tub extremely slippery and the patient might be injured. The patient must soak in the bathtub for 15 to 20 minutes three or four times a day. The bathtub should be filled with tepid water until it covers the affected area. The affected area should not be dried by rubbing with a towel but must be gently patted to avoid increased irritation and inflammation. p. 423

Which intervention would be most helpful in managing cellulitis in an 83-year-old patient with heart failure? A. Maintain strict bed rest B. Apply moist heat and elevation C. Soak the extremity in cool, sterile saline D. Perform passive range of motion exercises

B. Apply moist heat and elevation Rationale The application of warm, moist heat and elevation speeds the resolution of inflammation and infection when accompanied by appropriate antibiotic therapy. It does this by increasing local circulation to the affected area to bring macrophages to the area and carry off cellular debris. Strict bed rest is not advised, because this will place the patient at an increased risk for blood clot formation. Passive range of motion may be performed; however, this will not alter the course of the cellulitis. The extremity should not be soaked in cool saline because of vasoconstriction and possible delayed wound healing. Test-Taking Tip: Choose the best answer for questions asking for a single answer. More than one answer may be correct, but one answer may contain more information or more important information than another answer. p. 414

A mother and her child have been diagnosed with scabies after attending a camp together. What is an appropriate measure in treating this condition? A. Topical application of griseofulvin B. Applying 5% permethrin to the body C. Moist compresses applied frequently D. Administration of systemic antibiotics

B. Applying 5% permethrin to the body Rationale Scabies is treated with 5% permethrin topical lotion, one overnight application with a second application one week later. Scabies is not treated with griseofulvin. The patient should be taught to keep the area dry. Systemic antibiotics should only be used if secondary infections are present. STUDY TIP: Enhance your time-management abilities by designing a study program that best suits your needs and current daily routines by considering issues such as the following: (1) Amount of time needed; (2) Amount of time available; (3) "Best" time to study; (4) Time for emergencies and relaxation. p. 416

A nurse is teaching a patient about self-examination of skin lesions. The skin should be examined for which characteristics? Select all that apply. A. Depth of the lesion B. Asymmetry of the lesion C. Temperature of the lesion D. Irregularity of the borders of the lesion E. A change or evolution in appearance of the lesion

B. Asymmetry of the lesion D. Irregularity of the borders of the lesion E. A change or evolution in appearance of the lesion Rationale Self-examination of skin lesions is done by the ABCDE rule: A, Asymmetry; B, Border irregularity, C, Color change; D, Diameter of 6 mm or more; and E, Evolving in appearance. Depth and temperature of the lesion may be difficult to determine in a self-examination, and these characteristics are not included in the ABCDE rule. p. 410

In teaching a patient with basal cell carcinoma (BCC) about this disorder, the nurse considers that which statement about this skin cancer is true? A. BCC is the deadliest type of skin cancer. B. BCC is the most common type of skin cancer. C. Prognosis depends upon the thickness of the lesion. D. The cancerous cells of BCC usually spread beyond the skin.

B. BCC is the most common type of skin cancer. Rationale BCC is a locally invasive malignancy arising from epidermal basal cells. It is the most common type of skin cancer and also the least deadly. The cancerous cells of BCC almost never spread beyond the skin. BCC is the least deadly type of skin cancer. Prognosis depends on other factors too, not just the thickness of the lesion. BCC does not generally spread beyond the skin. p. 411

A patient has chills, malaise, and a body temperature of 100° F. Further assessment reveals a hot, tender, reddened, edematous area with a diffuse border on one leg. Which skin infection does the nurse suspect the patient initially had? A. Impetigo B. Cellulitis C. Carbuncle D. Furunculosis

B. Cellulitis Rationale Cellulitis is marked by a hot, tender, reddened, edematous area with a diffuse border resulting from inflammation of subcutaneous tissue. Untreated cellulitis may progress to gangrene. A patient who has pustules with thick, honey-colored crusting may have impetigo, which, if left untreated, may cause glomerulonephritis. A patient with many pustules in an erythematous area may have carbuncle. Furunculosis is mostly seen in patients who are obese, diabetic, and chronically ill. The affected patient may have a tender, erythematous area around a hair follicle, along with regional adenopathy and increased body temperature. p. 414

Which therapy uses subfreezing temperatures to destroy epidermal lesions? A. Excision B. Cryosurgery C. Punch biopsy D. Electrodesiccation

B. Cryosurgery Rationale Cryosurgery uses subfreezing temperatures to remove epidermal lesions. Excision is the removal of the tissue and is considered if the lesion to be removed involves the dermis. A punch biopsy is a dermatologic procedure used to obtain tissue samples for histologic study or to remove small lesions. Electrodesiccation involves the use of heat to destroy superficial tissue. STUDY TIP: Identify your problem areas that need attention. Do not waste time on restudying information you know. p. 422

Which surgical therapy can only be used to remove small, soft skin tumors and superficial lesions? A. Excision B. Curettage C. Skin scraping D. Punch biopsy

B. Curettage Rationale Curettage is the removal or scooping of tissue and is useful for removing many types of small, soft skin tumors and superficial lesions such as warts, actinic keratoses, and small basal and squamous cell carcinomas. Excision is the complete removal of a lesion and is considered only if the lesion to be removed involves the dermis. Performed with a scalpel blade, a skin scraping is used to obtain a sample of surface cells for microscopic inspection and diagnosis. A punch biopsy is used to obtain a tissue sample for histologic study or to remove small lesions. p. 422

A patient diagnosed with actinic keratosis is prescribed fluorouracil. To prevent side effects of the medication, what action should the nurse take? A. Administer an antihistamine. B. Instruct the patient to avoid sunlight. C. Advise the patient to use antipruritic lotion. D. Administer gamma-benzene hexachloride.

B. Instruct the patient to avoid sunlight. Rationale Fluorouracil is a topical cytotoxic agent. It is used for the treatment of actinic keratosis and is a photosensitizing drug. Patients should be instructed to avoid sunlight during treatment. Antipruritic lotion is used to relieve itching in cases of insect bites. Gamma-benzene hexachloride is used to treat pediculosis. Antihistamines are used to prevent allergic reactions. p. 421

A patient is diagnosed with vaginal candidiasis. What nursing interventions are appropriate for this patient? Select all that apply. A. Administer antibiotics as advised. B. Instruct the patient to keep the area clean and dry. C. Instruct the patient to avoid any vaginal suppositories. D. Inform the patient to abstain from sex or use condoms. E. Instruct the patient to use antifungal powder on the affected skin.

B. Instruct the patient to keep the area clean and dry. D. Inform the patient to abstain from sex or use condoms. E. Instruct the patient to use antifungal powder on the affected skin. Rationale Candidiasis is a condition caused by the fungus Candida albicans. Candida thrives in warm, moist areas. The nurse should instruct the patient to maintain good personal hygiene and keep the affected area clean and dry. The patient should be instructed to abstain from sex or use condoms until the candidiasis is resolved to prevent worsening symptoms. Antifungal powders can be used to prevent the recurrence of the fungal infection. Antibiotics are not used, because they are ineffective against Candida. The use of vaginal suppositories may be advised, and some treatments for candidiasis are in the vaginal suppository form. p. 418

When a patient has psoriasis, which sign would a nurse expect to find? A. Pustules in skinfolds B. Itchy, scaly patches on scalp C. Macular rash on the trunk area D. Vesicular rash on the extremities

B. Itchy, scaly patches on scalp Rationale Psoriasis is a noncontagious autoimmune disease. It usually presents as itchy scales on the scalp, knees, or elbows, the result of an overproduction of skin cells. Other commonly affected areas are the joints, fingernails, and toenails. Psoriasis does not present as pustules, as a macular rash, or as a vesicular rash. p. 419

A patient is recovering from incision and drainage for a furuncle. What action should the nurse take when caring for this patient? A. Clean the oozing wound with soap and plain water. B. Leave the crust that forms over the damaged area undisturbed. C. Keep the wound dry and covered with a dressing for rapid healing. D. Use antibiotic ointment with a dressing that is both absorbent and adherent.

B. Leave the crust that forms over the damaged area undisturbed. Rationale The initial crust that forms over the damaged area must be left undisturbed because the crust serves as a protective coating for the damaged skin beneath it. The oozing wound should be cleansed with saline solution twice a day. Wounds that are kept moist and covered heal rapidly without scarring; the wound should not be kept dry. An antibiotic dressing that is both absorbent and nonadherent should be used. p. 414

The nurse assesses an eroded, ulcerative growth less than 1 cm wide on a patient's chest. The lesion is flat and variegated in color. What condition should be suspected related to this assessment finding? A. Basal cell carcinoma B. Malignant melanoma C. Squamous cell carcinoma D. Cutaneous T-cell lymphoma

B. Malignant melanoma Rationale Malignant melanoma is characterized by variegated color, including red, white, blue, black, gray, and brown. The growth may be flat or elevated; it is eroded or ulcerated and usually less than 1 cm. Basal cell carcinoma is characterized by a small, slowly enlarging papule with semitranslucent borders and overlying telangiectasia. Squamous cell carcinoma is marked by a thin, scaly, erythematous plaque that does not invade the dermis. Cutaneous T-cell lymphoma involves three stages; a patch is the characteristic feature of the early stage, and tumors are observed in the advanced stage. STUDY TIP: Establish your study priorities and the goals by which to achieve them. Write them out and review the goals during each of your study periods to focus your preparation efforts. p. 411

A patient tells a nurse, "I think I might have head lice." Which assessment findings would the nurse observe with this infestation? A. Diffuse pruritic wheals B. Oval white dots stuck to hair shafts C. Pruritic papules with linear burrows at the hairline D. Itchy redness and edema over the area of infestation

B. Oval white dots stuck to hair shafts Rationale The eggs of lice, known as nits, appear as oval white dots attached to hair shafts. The lice are not usually visible. Diffuse pruritic wheals may be seen with localized inflammation such as that in response to an insect bite. Itchy redness and edema over the area of infestation are not characteristic of head lice. Pruritic papules with linear burrows at the hairline are characteristic of scabies. p. 416

Which base used for topical agents produces a drying effect of the skin? A. Gel B. Paste C. Cream D. Ointment

B. Paste Rationale A paste is a mixture of ointment and powder. The moisture from the paste is absorbed into the skin, which dries the affected area. A gel is a nongreasy combination of propylene glycol and water; some gels may also contain alcohol. A cream is an emulsion of oil and water used for lubrication and protection. An ointment is a combination of oil and water, useful for lubrication and for prevention of dehydration. p. 423

The nurse is caring for four patients in a dermatology unit. Which patient may be treated with liquid nitrogen therapy? A. Patient A: Cheesy white plaque, resembling milks curds, in the mouth: candida albicans B. Patient B: Circumscribed flesh clored papule thats growing and painful when its sides are pressed: Human papilloma virus C. Patient C: Interdigital scaling and maceration, along with erythema, on scaly plantar surfaces: Dermatophyte D. Patient D: Distrubution of pustules along a specific line on the left side of the stomach with redness of the area, a burning sensation, and pain: Varicella zoster

B. Patient B: Circumscribed flesh clored papule thats growing and painful when its sides are pressed: Human papilloma virus Rationale Patient B has a circumscribed, hypertrophic, flesh-colored papule that is painful on lateral compression; this is the clinical manifestation of verruca vulgaris caused by human papillomavirus. Liquid nitrogen therapy, which is used to remove abnormal growths from the skin, will be beneficial. Patient A has white cheesy plaques, resembling milk curds, in the mouth; this indicates candidiasis caused by Candida albicans. Candidiasis requires treatment with azole antifungals such as fluconazole and ketoconazole. Patient C has interdigital scaling and maceration, along with erythema, on scaly plantar surfaces; these are signs of tinea pedis caused by a dermatophyte. Treatment includes topical antifungal cream, solution, gel, or spray. Patient D has a unilateral distribution of vesicles along a dermatome on an erythematous base accompanied by burning and pain. This is the clinical appearance of herpes zoster (shingles), caused by varicella zoster. Antiviral agents such as famciclovir and acyclovir are recommended for the treatment of herpes zoster (shingles). Test-Taking Tip: Identify the type of skin conditions from the findings presented for each patient and recollect the treatment options for those conditions. Remembering the pharmacologic class of the drug mentioned in the question stem may also help you choose the right option. p. 415

A patient complains of excessive itching along the abdomen near the umbilicus. What instructions should the nurse give to the patient to relieve pruritus? Select all that apply. A. Rub the affected area gently. B. Place moist cotton sheets on the affected area. C. Warm the affected area with warm compresses. D. Apply menthol or camphor locally to the affected area. E. Use good-quality moisturizers on the affected area after drying it.

B. Place moist cotton sheets on the affected area. D. Apply menthol or camphor locally to the affected area. E. Use good-quality moisturizers on the affected area after drying it. Rationale Pruritus, or severe itching, needs to be controlled because an excoriated lesion may be difficult to assess. A variety of measures can be used to control pruritus. The pruritic area should always be moist because dry skin increases the itch sensation. When applied topically, menthol and camphor numb the itch receptors and thus decrease itch sensation. Applying moisturizers and moist linen can be helpful in controlling itch. Heat and rubbing the affected area causes vasodilation, which worsens itching. p. 424

A nurse is assessing a patient diagnosed with malignant melanoma. The nurse understands that the prognosis of the cancer can be assessed by using the Breslow's measurement. How is the prognosis related to Breslow's measurement? A. The larger the tumor, the worse the prognosis B. The deeper the tumor, the worse the prognosis C. The darker the tumor, the worse the prognosis D. The greater number of tumors, the worse the prognosis

B. The deeper the tumor, the worse the prognosis Rationale Tumor thickness is an important prognostic factor for melanoma. The Breslow's measurement is used to assess the depth of the tumor in millimeters. The deeper the tumor, the worse will be the prognosis of melanoma. Size of the tumor, color of the tumor, and an increase in the number of tumors are not determined using the Breslow's measurement. pp. 411-413

Which patient does the nurse suspect is most at risk for contracting Lyme disease? A. The patient bitten by lice B. The patient bitten by ticks C. The patient bitten by mites D. The patient stung by wasps

B. The patient bitten by ticks Rationale Borrelia burgdorferi is a spirochete transmitted by ticks in certain geographic areas; the person bitten by a tick is at risk for Lyme disease. A person who is bitten by lice such as Pediculus humanus var. capitis, Pediculus humanus var. corporis, or Phthirus pubis is at risk for pediculosis. A person who is bitten by a mite such as Sarcoptes scabiei may have scabies. A person who is stung by a wasp of the Hymenoptera species may experience a severe hypersensitivity reaction. p. 416

A patient is being treated for scaly skin lesions. The results of a potassium hydroxide (KOH) test that was performed using the scrapings of the lesions are positive. How should the nurse interpret the result? A. The patient has scabies. B. The patient has a fungal infection. C. The patient has contact dermatitis. D. The patient has basal cell carcinoma.

B. The patient has a fungal infection. Rationale A potassium hydroxide (KOH) test involves microscopic examination of scrapings of scaly skin lesions in a 10% to 20% KOH solution. A positive KOH test indicates that the patient has a fungal infection of the skin. A basal cell carcinoma is confirmed through a biopsy examination. Contact dermatitis is confirmed through a patch test. Presence of scabies is confirmed through a mineral oil test. p. 417

Which patient does the nurse determine will benefit the most from treatment with methotrexate? A. The patient with a benign adipose tumor B. The patient with autoimmune chronic dermatitis C. The patient with inflammatory disorder of sebaceous glands D. The patient with an increase in normal melanocytes in the basal layer of epidermis

B. The patient with autoimmune chronic dermatitis Rationale The skin condition associated with autoimmune chronic dermatitis is psoriasis, which involves excessively rapid turnover of the cells. Methotrexate slows down the rapid division of skin cells that is a hallmark of psoriasis. A benign tumor of adipose tissue is called a lipoma; it generally does not require treatment but excision is the usual treatment option. An inflammatory disorder of sebaceous glands is acne vulgaris, usually treated with topical benzoyl peroxide or other antimicrobials. An increased number of normal melanocytes in the basal layer of epidermis is called lentigo. Treatment options include liquid nitrogen and laser resurfacing. p. 419

Which assessment finding of a 70-year-old male patient's skin should the nurse prioritize? A. The patient's complaint of dry skin that is frequently itchy. B. The presence of an irregularly shaped mole that the patient states is new. C. The presence of veins on the back of the patient's leg that are blue and tortuous. D. The presence of a rash on the patient's hand and forearm to which the patient applies a corticosteroid ointment.

B. The presence of an irregularly shaped mole that the patient states is new. Rationale Although dry skin that is frequently itchy, veins on the back of the patient's leg, and a rash on the patient's hand and forearm are significant, the presence of an irregular mole that is new is suggestive of a neoplasm and warrants immediate follow-up. Test-Taking Tip: On a test day, eat a normal meal before going to school. If the test is late in the morning, take a high-powered snack with you to eat 20 minutes before the examination. The brain works best when it has the glucose necessary for cellular function. p. 410

A nurse is caring for a patient diagnosed with shingles. The primary health care provider prescribes acyclovir to be administered as soon as possible. The most likely reason for the medication is to prevent what? A. To prevent pain B. To prevent postherpetic neuralgia C. To prevent worsening of symptoms D. To prevent the patient from getting restless

B. To prevent postherpetic neuralgia Rationale Following the onset of symptoms of shingles (herpes zoster), antiviral agents such as acyclovir should be administered within 72 hours to prevent postherpetic neuralgia. Analgesics are given to relieve pain and mild sedatives to prevent restlessness. Symptomatic treatment is given to prevent worsening of symptoms. p. 415

A nurse is caring for a patient with quadriplegia who is confined to bed. The patient shows signs of skin irritation. Which measures are appropriate for preventing further skin damage when bathing the patient? Select all that apply. A. Use hot water for baths. B. Use lipid-free cleansers. C. Scrub the skin very lightly. D. Use soaps having a high acid content. E. Apply a moisturizer after wiping the skin dry with a towel.

B. Use lipid-free cleansers. C. Scrub the skin very lightly. Rationale The patient shows signs of skin irritation, so the skin should be scrubbed lightly. Vigorous scrubbing may increase skin irritation. Use of mild, lipid-free cleansers can also reduce skin irritation. Hot water can increase the dryness of the skin, making it more prone to irritation. Soaps having high acid content can also cause skin irritation and are therefore not recommended for this patient. Moisturizers should always be applied on damp skin so that the moisture is sealed. p. 409

Which laboratory result is the best indicator that a patient with cellulitis is recovering from this infection? A. White blood cells (WBC) of 2900/µL B. White blood cells (WBC) of 8200/µL C. White blood cells (WBC) of 12,700/µL D. White blood cells (WBC) of 16,300/µL

B. White blood cells (WBC) of 8200/µL Rationale The normal WBC count is generally 4000 to 11,000/µL. For this reason, the patient's level would be returning to normal if it were 8200/µL, indicating recovery from cellulitis. The 2900/µL is too low and indicates that another problem is occurring. WBCs of 12,700/µL and 16,300/µL are evidence of continuing infection. Test-Taking Tip: Stay away from other nervous students before the test. Stop reviewing at least 30 minutes before the test. Take a walk, go to the library and read a magazine, listen to music, or do something else that is relaxing. Go to the test room a few minutes before class time so that you are not rushed in settling down in your seat. Tune out what others are saying. Crowd tension is contagious, so stay away from it. p. 414

When studying the incidence of skin cancers in a population, a nurse finds that a greater number of skin cancer cases have been reported in white patients than in African American patients. What could be the most likely cause of such an occurrence? A. Whites usually have less exposure to the sun than African Americans. B. Whites have less melanin content in their skin than African Americans. C. Whites usually have more exposure to the sun than African Americans. D. Whites have greater melanin content in their skin than African Americans.

B. Whites have less melanin content in their skin than African Americans. Rationale Melanin provides natural protection to the skin against the harmful radiation of the sun. Therefore melanin plays a major role in preventing skin cancer. African American people are darker skinned and have greater melanin content in their skin. Therefore they are less susceptible to skin cancer than white people. White people are lighter skinned and have less melanin. Differences in sun exposure between the two populations cannot be predicted. p. 410

The nurse is caring for patients who are experiencing side effects of different cosmetic topical procedures. Which patient may have been treated for fine wrinkling? A. A patient with severe redness, flaking, and oozing at the treated area B. A patient who has photosensitivity and a pink tone that lasts for 24 hours C. A patient with redness and inflammation and increase in the pigmentation D. A patient who is experiencing photosensitivity and has redness lasting for seven weeks

C. A patient with redness and inflammation and increase in the pigmentation Rationale Tretinoin is used in the treatment of fine wrinkling; its side effects include erythema, swelling, and pigmentation changes. A patient with severe erythema, oozing, and flaking of the skin may have been treated with α-hydroxy acids at high concentrations. A patient with photosensitivity and a pink tone that resolves within 24 hours may have been treated with microdermabrasion. A patient experiencing photosensitivity and erythema that lasts from six to eight weeks may have been treated with chemical peels. p. 426

A patient is diagnosed with folliculitis after frequent use of a hot tub. About what treatment options should the nurse educate the patient? Select all that apply. A. Oral penicillin B. Incision and drainage C. Antistaphylococcal soap D. Warm compress of aluminum acetate E. Immobilization and elevation of the affected area

C. Antistaphylococcal soap D. Warm compress of aluminum acetate Rationale Folliculitis is characterized by small pustules at the hair follicle openings and minimal erythema. Antistaphylococcal soap-and-water cleansing is used to treat folliculitis. Warm compresses of water or aluminum acetate solution are also used as treatment. Systemic antibiotics, such as oral penicillin, are used in the treatment of impetigo, furunculosis, and cellulitis. Incision and drainage is used to treat furuncle and furunculosis. Immobilization and elevation of the affected area is appropriate for patients with cellulitis. Test-Taking Tip: Identify option components as correct or incorrect. This may help you identify incorrect answers. p. 414

A patient presents with intense pain and pruritus from a recent bee sting. What is the most appropriate nursing action? A. Apply moisturizers. B. Apply warm compresses. C. Apply a local antipruritic lotion. D. Administer antibiotics as prescribed.

C. Apply a local antipruritic lotion. Rationale Bee stings may cause intense pain and pruritus. These symptoms are usually relieved by a local application of antipruritic lotions. Cold compresses can be given to soothe the burning sensation. Antihistamines can be administered. Antibiotics are not indicated because there is no evidence of infection. Moisturizers are not helpful in relieving the symptoms. Test-Taking Tip: Prepare for exams when and where you are most alert and able to concentrate. If you are most alert at night, study at night. If you are most alert at 2 AM, study in the early morning hours. Study where you can focus your attention and avoid distractions. This may be in the library or in a quiet corner of your home. The key point is to keep on doing what is working for you. If you are distracted or falling asleep, you may want to change when and where you are studying. p. 424

The nurse is caring for a patient with a skin suture. How should the nurse care for this wound? A. Clean daily with saline solution. B. Cover with a wet sterile dressing. C. Apply an antibiotic and leave open to air. D. Administer diphenhydramine to treat inflammation.

C. Apply an antibiotic and leave open to air. Rationale The best method of caring for a skin suture involves using a topical antibiotic and exposing the wound to the air. The wound should be covered with a dry sterile dressing, not a wet dressing. The incision lines must be cleaned every day with plain tap water. Any inflammation or sign of infection should be reported to the health care provider, who would advise the patient whether to use antihistamines. p. 424

What condition does the nurse suspect in an asthmatic patient who reports erythema, oozing vesicles, and severe itching of the skin? A. Urticaria B. Drug reaction C. Atopic dermatitis D. Allergic contact dermatitis

C. Atopic dermatitis Rationale Asthma and atopic dermatitis are atopic diseases with the same pathogenic base with regard to allergic reaction and type of oversensitivity leading to allergic inflammation. Therefore atopic dermatitis may be associated with asthma. Urticaria is an allergic reaction associated with erythema and edema in the upper epidermis. A drug reaction is caused by any drug that acts as an antigen and causes a hypersensitivity reaction. Allergic contact dermatitis is a manifestation of delayed hypersensitivity reaction, which results from sensitization after one or more exposures. It is characterized by the appearance of lesions two to seven days after contact with an allergen. p. 418

About which treatments should the nurse educate the patient with scaly, ringlike lesions with well-defined margins on both arms? Select all that apply. A. Cryosurgery B. Oral penicillin C. Cool compress D. Mupirocin cream E. Miconazole cream

C. Cool compress E. Miconazole cream Rationale Scaly ringlike lesions with well-defined margins on the patient's arms are indicative of tinea corporis. The treatment of tinea corporis includes the use of cool compresses and creams or solutions of such antifungal drugs such as miconazole or ketoconazole. Cryosurgery is performed to remove plantar warts from the bottom of the foot. Oral penicillin, which is an antibiotic, is used to treat infections such as impetigo that are caused by bacteria. Creams such as mupirocin, which is an antibiotic, are used to treat skin conditions such as impetigo and folliculitis that are caused by bacteria. p. 418

Which nursing intervention would be most effective in improving the comfort of a patient with herpes zoster? A. Direct sunlight B. Dry heating pad C. Cool, wet dressing D. Warm, moist compress

C. Cool, wet dressing Rationale Application of a cool, wet dressing followed by an analgesic is usually effective in relieving discomforts related to herpes zoster (shingles). The application of any form of warmth may increase the sensitivity of nerve endings and worsen the pain. p. 415

A patient is experiencing atrophy of the skin of the forearm that started after three weeks of treatment. Which therapy should the nurse suspect the patient is undergoing? A. Mupirocin therapy B. Hydroxyzine therapy C. Corticosteroid therapy D. Topical fluorouracil therapy

C. Corticosteroid therapy Rationale Patients who are being treated with high-potency corticosteroids may experience side effects such as a skin atrophy, which is caused by impaired cell mitosis. This effect develops after two to three weeks of corticosteroid use. Mupirocin is a topical antibiotic and patients who are allergic may develop contact dermatitis. Hydroxyzine is an antihistamine that produces sedation. Topical fluorouracil is a cytotoxic agent that produces erythema and pruritus within three to five days. p. 421

The nurse is educating a patient regarding complications due to diabetes. Which skin infection's incidence is increased in patients with diabetes mellitus? A. Cellulitis B. Impetigo C. Folliculitis D. Furunculosis

C. Folliculitis Rationale The incidence of folliculitis is increased among people with diabetes mellitus. Cellulitis is typically a secondary complication of a disease but may occur as a primary infection. Impetigo, typically associated with poor hygiene, may be a primary or secondary infection. The incidence of furunculosis is increased among people with obesity, diabetes, and chronic illness and those who are regularly exposed to moisture or pressure. p. 414

A 54-year-old patient with diabetes mellitus has cellulitis of the right lower extremity. Which assessment finding would the nurse expect on physical examination? A. Pallor of the right toes B. Delayed capillary refill time C. Hot, tender edematous area D. Paresthesias of the right lower extremity

C. Hot, tender edematous area Rationale Cellulitis is a diffuse, acute infection of the skin. It is characterized by redness and hot, tender edematous area. These changes accompany the processes of inflammation and infection. Delayed capillary refill time and pallor of the toes would indicate a circulatory impairment, not infection. Paresthesias would be indicative of poorly controlled diabetes. p. 414

The nurse assesses a vesiculopustular lesion with a thick, honey-colored crust surrounded by erythema on the left side of a patient's arm. The patient states that he or she had a few bumps in the area that the patient had been scratching. What complication has the patient developed from this condition? A. Cellulitis B. Furuncle C. Impetigo D. Folliculitis

C. Impetigo Rationale Impetigo is marked by vesiculopustular lesions that develop a thick, honey-colored crust surrounded by erythema. They are most common on the face as a primary infection. Cellulitis manifests clinically as a hot, tender, erythematous, edematous area with a diffuse border. Furuncle is marked by a tender erythematous area around a hair follicle. Small pustules at the hair follicle opening with minimal erythema and development of crusting are indicative of folliculitis. STUDY TIP: Enhance your organizational skills by developing a checklist and finding ways to improve your ability to retain information—for instance, using index cards with essential data, which are easy to carry and review whenever you have a spare moment. p. 414

During an admission assessment, the nurse decides to implement contact precautions if which of these disorders is present in a patient? A. Psoriasis B. Tinea unguium C. Impetigo on the lower legs D. Candidiasis in the groin area

C. Impetigo on the lower legs Rationale Impetigo is caused by a bacterial infection (group A β-hemolytic streptococci or staphylococci) and is highly contagious. Good skin hygiene and infection control practices are necessary to prevent the spread of this infection. Psoriasis is an autoimmune chronic dermatitis and is not contagious. Tinea unguium is a fungal infection of the nails. Candidiasis is a fungal infection in the skin folds. p. 418

The nurse, preparing educational information about types of skin cancer, recalls that which type has a higher risk for metastasis and poor prognosis unless it is treated early? A. Myeloma B. Basal cell C. Melanoma D. Squamous cell

C. Melanoma Rationale Malignant melanoma has a higher risk for metastasis than does basal cell or squamous cell carcinoma because of its invasiveness. There is a poor prognosis unless melanoma is diagnosed and treated early. Basal cell and squamous cell carcinoma are typically superficial, localized lesions that respond well to treatment. Myeloma is a hematologic malignancy, not a skin cancer. p. 411

A nurse is assessing an obese patient with skin that is dry and irritated. The nurse understands that the most likely reason the patient is experiencing the skin condition is what? A. Obese people drink less water. B. Obese people have lowered immunity. C. Obese people are prone to overheating and sweating. D. Obese people have improved arterial and venous flow.

C. Obese people are prone to overheating and sweating. Rationale Obesity increases the risk of skin inflammation and dryness. Increased subcutaneous fat can lead to overheating and increased sweating. Increased sweating may lead to skin inflammation and dryness. Obese people are not known to drink less water. Lowered immunity does not lead to skin inflammation and dryness. Obese people tend to have an impaired arterial and venous flow; however, this does not inflame or dry the skin. p. 409

The nurse is caring for patients who have undergone various treatments. According to the chart, which patient may have squamous cell carcinoma? A. Patient A: The patient develops cataracts after direct exposure to the sun. B. Patient B: The patient has weak, wrinkled, and thin skin, but the tissue beneath is healthy with red eruptions and severe acne vulgaris. C. Patient C: The skin became wrinkled and thin along with the development of dark pigmentation in the area treated. The patient also developed hearing loss and ocular damage. D. Patient D: The patient develops redness and itching after 4 days of treatment and has eroded the areas over the damaged skin, which hurts in the second week of the treatment.

C. Patient C: The skin became wrinkled and thin along with the development of dark pigmentation in the area treated. The patient also developed hearing loss and ocular damage. Rationale Patient C has atrophy of the skin with hyperpigmentation in the treated area, hearing loss, and ocular damage, all of which are adverse effects of radiation therapy, which is used in the treatment of basal and squamous cell carcinoma. Patient A has developed cataracts on exposure to the sun and thus may have taken psoralen, which is used in phototherapy. Phototherapy is used to treat actinic keratosis and malignant skin tumors. Patient B has atrophy of the upper layer of the skin with red eruptions and acne vulgaris, which are adverse effects of corticosteroid use. Patient D has developed erythema and pruritus within four days of treatment and painful, eroded areas of damaged skin within two weeks of the treatment; these are adverse effects of topical fluorouracil. Test-Taking Tip: Each treatment for skin problems has different adverse effects. Identify them and relate them to the skin condition for which the treatment is done. p. 410

A nurse is performing a skin assessment for an obese patient. The nurse suspects the patient has diabetes mellitus. Which skin conditions indicate diabetes mellitus? Select all that apply. A. Petechiae B. Skin desquamation C. Presence of skin tags D. Seborrhea-like lesions E. Rashes under the breast and axillae

C. Presence of skin tags E. Rashes under the breast and axillae Rationale Obese people have a high risk of developing type II diabetes mellitus. The presence of skin tags and rashes under the breast and axillae are signs of diabetes mellitus in obese people. Petechiae, skin desquamation, and seborrhea-like lesions are not seen in diabetic patients. Petechiae are minor hemorrhages characterized by small purple spots under the skin; petechiae are usually a sign of vitamin C deficiency. Skin desquamation may occur due to skin irritation. Seborrhea-like skin lesions indicate vitamin B deficiency. p. 419

A nurse is caring for a patient with liver disease that is scheduled for psoralen plus UVA therapy (PUVA). The nurse understands that the reason that extreme caution should be exercised while administering the therapy to the patient is what? A. Psoralen will excrete slowly in the patient. B. Psoralen will cause erythema in the patient. C. Psoralen will prolong photosensitivity in the patient. D. Psoralen will cause nausea and vomiting in the patient.

C. Psoralen will prolong photosensitivity in the patient. Rationale In psoralen plus UVA light (PUVA) therapy, a photosensitizing drug called psoralen is given to patients for a prescribed amount of time before exposure to UVA light. The drug should be administered with extreme caution in patients having liver disease. Liver disease tends to slow down metabolism, and this may lead to prolonged photosensitivity. Erythema is a side effect of phototherapy and is not accentuated by psoralen. Psoralen excretion may be slower in patients with impaired renal function. It is not the reason for taking special precautions in the patient. Nausea and vomiting are common side effects of psoralen, not specifically related to liver impairment, and they do not call for special precautions. p. 420

A patient presents with tiny skin lesions on the chest and back that have a diameter less than 0.5 cm. Which procedure should be used to obtain a sample of these lesions? A. Curettage B. Skin scraping C. Punch biopsy D. Electrodessication

C. Punch biopsy Rationale Punch biopsy is a common dermatologic procedure used to obtain a tissue sample for histologic study. It is generally reserved for lesions smaller than 0.5 cm. Curettage is useful for removing soft skin tumors and superficial lesions, such as warts and actinic keratoses. Skin scraping is done to obtain a sample of surface cells for microscopic inspection and diagnosis. Electrodessication is useful in coagulation of bleeding vessels to obtain hemostasis and destruction of small telangiectasias. p. 422

A 56-year-old white patient presents with a flat, dry, scaly area on the eyebrows that is treated with a chemical peel. What should the nurse include in the discharge teaching? A. Metastasis of this type of cancer is rare. B. The patient has an increased risk for melanoma. C. Recurrence of the premalignant lesion is possible. D. Untreated lesions may metastasize to regional lymph nodes.

C. Recurrence of the premalignant lesion is possible. Rationale The flat or elevated dry scaly area is actinic keratosis from sun damage and is a premalignant skin lesion common in older whites, with possible recurrence even with adequate treatment. Metastasis of basal cell carcinoma is rare; it is a small, slowly enlarging papule. There is an increased risk for melanoma with atypical or dysplastic nevi. With squamous cell carcinoma, untreated lesions may metastasize to regional lymph nodes and distant organs, but it has a high cure rate with early detection and treatment. p. 411

Which does the nurse suspect in a patient who reports severe nighttime body itching and has red, crusted papules? A. Lice B. Ticks C. Scabies D. Bedbugs

C. Scabies Rationale Severe nighttime itching and red papules are indicative of scabies. Lice bites result in minute, red noninflamed points that progress to a papular wheal-like lesion. A tick bite that transmits Lyme disease is characterized by a spreading ringlike rash that occurs three to four weeks after the bite. A bedbug bite results in a wheal surrounded by a vivid flare and firm urticaria that transforms into a persistent lesion. p. 416

When teaching sun safety guidelines, what instructions should the nurse include when teaching about sunscreen lotion and creams? Select all that apply. A. Sunscreens are not required in cloudy weather. B. Sunscreens should be reapplied after six hours. C. Sunscreens should be reapplied immediately after swimming. D. Sunscreens should have a minimum sun protection factor (SPF) of 15. E. Sunscreens should be applied 20 to 30 minutes before going outdoors.

C. Sunscreens should be reapplied immediately after swimming. D. Sunscreens should have a minimum sun protection factor (SPF) of 15. E. Sunscreens should be applied 20 to 30 minutes before going outdoors. Rationale Sunscreens are creams and lotions that filter both ultraviolet A and ultraviolet B rays and can prevent dermatologic problems. When choosing a sunscreen, the patient should consider one with SPF 15. A patient with a family or personal history of melanoma should be advised to use a sunscreen with SPF 30. Sunscreen should be applied on the skin 20 to 30 minutes before going outdoors. Though many sunscreens are waterproof, they should be reapplied immediately after swimming in case the sunscreen is diluted in the water or rubbed off. Sunscreen should be applied even in cloudy weather, because the ultraviolet rays can penetrate clouds. Because the effect of a sunscreen decreases with time, it should be reapplied every two hours. pp. 407-408

A middle-aged obese patient presents with small, skin-colored, soft, pedunculated papules on the neck, axillae, and upper trunk. How should the nurse interpret these findings? A. The patient has nevi. B. The patient has lentigo. C. The patient has acrochordons. D. The patient has seborrheic keratoses.

C. The patient has acrochordons. Rationale Small, skin-colored, soft, pedunculated papules are a manifestation of acrochordons. They are also called skin tags, and are commonly found on the neck, axillae, and upper trunk. They are mostly found in obese patients during middle age. Nevi or moles are hyperpigmented areas that vary in form and color. Lentigo is hyperpigmented, brown to black maculae or patches on sun-exposed areas. Seborrheic keratoses are characterized by irregular verrucous papules or plaques that are well-defined in shape. p. 419

Which fungal infection manifests on the surface of a patient's skin with an erythematous and typical annular scaly appearance and well-defined margins? A. Tinea pedis B. Tinea cruris C. Tinea corporis D. Tinea unguium

C. Tinea corporis Rationale Tinea corporis is commonly referred to as ringworm. Tinea corporis infection has an erythematous, annular (ringlike) scaly appearance with well-defined margins. A tinea pedis fungal infection is characterized by scaly plantar surfaces that are pruritic and blistering in nature. Tinea cruris infection does not affect mucous membranes, and it is associated with well-defined scaly plaque on the patient's groin area. A patient with tinea unguium infection has brittle, thickened, and broken nails with yellowish discoloration. Test-Taking Tip: Identify option components as correct or incorrect. This may help you identify a wrong answer. p. 418

A patient complains of frequently having bleeding gums as well as widespread bruising and small red and purple dots all over the body. The patient is most likely deficient in which vitamin? A. Vitamin A B. Vitamin B C. Vitamin C D. Vitamin K

C. Vitamin C Rationale Vitamin C is responsible for connective tissue building and wound healing. Its deficiency leads to a condition known as scurvy, which is characterized by petechiae, purpura, and bleeding gums. Vitamin A deficiency causes dryness of the conjunctiva and night blindness. Vitamin B deficiency usually manifests as dermatologic symptoms, such as erythema, bullae, and seborrhea-like lesions. Vitamin K deficiency causes decreased synthesis of clotting factors, and the primary manifestation is bruising. Test-Taking Tip: Survey the test before you start answering the questions. Plan how to complete the exam in the time allowed. Read the directions carefully and answer the questions you know for sure first. p. 409

The nurse is teaching safety to a patient who has been prescribed phototherapy. Which statements by the patient indicate that teaching has been effective? A. "I can be exposed to the sun any time except during sunrise." B. "I can be exposed to ultraviolet bulbs at home but cannot go outside." C. "I can remove my protective eyewear if the glass windows are closed." D. "I must wear protective eyewear that blocks 100 percent of the ultraviolet light when going out."

D. "I must wear protective eyewear that blocks 100 percent of the ultraviolet light when going out." Rationale The patient is undergoing phototherapy, which means that the patient is taking psoralen, a photosensitizing drug. Therefore the patient should wear any protective eyewear that blocks 100 percent of ultraviolet light when going outside, to prevent the sun's ultraviolet rays from affecting the eyes. Any exposure to the sun during the day may affect the patient's skin. The patient should not be exposed to ultraviolet radiation at home. The patient must wear protective eyewear even if the glass windows are closed because ultraviolet light can penetrate glass. p. 420

The nurse has been discussing sun safety guidelines with a patient who plans to go to the beach after recovery from surgery. Which of these statements by the patient reflects a need for further education? A. "I will apply sunscreen liberally and often." B. "I will avoid going to the beach during the midday hours." C. "I will wear a large-brimmed hat and sunglasses while I'm outside." D. "If the weather is overcast, I don't need to worry about wearing sunscreen."

D. "If the weather is overcast, I don't need to worry about wearing sunscreen." Rationale Even on overcast days, serious sunburn can occur because up to 80% of the sun's ultraviolet (UV) rays can penetrate the clouds. Sunscreen should be applied liberally, and often, especially after swimming. Avoid sun exposure during midday hours because during that time the sun's rays are the most dangerous. A large-brimmed hat and sunglasses also provide protection from the sun's rays. pp. 407-408

A 26-year-old patient tells the nurse that the patient is afraid to use the treatment recommended for psoriasis because the patient's mother had a lot of problems with all the creams she used to try to treat her psoriasis. How should the nurse respond to the patient? A. "You will only know if you try it and see." B. "You may need to get counseling to help you cope." C. "No treatment is medically necessary, but it can be removed." D. "Topical, light therapy, and systemic medications are now available."

D. "Topical, light therapy, and systemic medications are now available." Rationale Treatment of psoriasis usually involves a combination of strategies, including topical treatments, phototherapy, or systemic medications, including biologic drugs. Telling the patient that he or she will only know if he or she tries it or that the patient may need counseling is denying the patient's concern. Psoriasis is treated to manage the disease, because the patient may have a weakened immune system and be at risk for cardiovascular disease. p. 419

A patient describes having small, firm reddened raised lesions with flat, rough patches causing intense pruritus to the nurse. The nurse suspects contact dermatitis. What would be the nurse's next assessment? A. History of seasonal allergies B. Initiation of new medication C. Previous pruritic skin lesions D. Activities in past two to seven days

D. Activities in past two to seven days Rationale The patient's lesions are papules and plaques characteristic of contact dermatitis. The nurse should ask the patient about activities over the past two to seven days to identify potential allergens because contact dermatitis has a delayed onset. Even if an offending agent is not identified, the nurse can provide patient teaching about managing the pruritus and preventing infection by decreasing scratching. Seasonal allergies and new medications are more likely to cause urticaria than papules and plaque. The nurse should also ask about pruritic rashes in the past to determine potential illnesses that can cause dermatologic manifestations. p. 418

A patient shows the nurse how the skin around an abdominal dressing is red and states that it itches. The nurse identifies an area of red papules with occasional papules that matches the area that had been taped around the dressing. The nurse suspects that the patient has which skin condition? A. Urticaria B. Tinea corporis C. Atopic dermatitis D. Allergic contact dermatitis

D. Allergic contact dermatitis Rationale Allergic contact dermatitis is a manifestation of delayed hypersensitivity characterized by red papules and plaques, and also is circumscribed sharply with occasional vesicles. It is usually pruritic. The area of dermatitis frequently takes the shape of the causative agent. Urticaria is spontaneously occurring, with raised or irregularly shaped wheals, varying size, and usually multiple in number. Tinea corporis is a fungal infection of the skin also known as ringworm. STUDY TIP: Develop a realistic plan of study. Do not set rigid, unrealistic goals. p. 418

Which practice would the nurse teach a patient when applying topical medication in the form of a paste? A. Avoid applying medications directly on to dressings. B. Use a tongue blade whenever the patient's skin integrity allows. C. Avoid covering skin regions that have topical medication in place. D. Apply a layer of medication that is just thick enough to ensure coverage.

D. Apply a layer of medication that is just thick enough to ensure coverage. Rationale Patients should be directed to avoid applying topical medications, including pastes, too thickly. Paste medications may be applied directly on to dressings, and regions with medications may be covered. A tongue blade is not normally used for the application of a thin coat of paste because it becomes difficult to spread evenly. p. 421

A patient has an itchy and painful rash on the anterior surface of both arms three days after coming into contact with poison ivy. To promote comfort, the nurse should suggest which nonpharmacologic intervention? A. Bathe in a tub of lukewarm water B. Soak the affected areas in kitchen vinegar C. Cover the affected areas with dry dressings D. Apply cool tap water compresses to the affected areas

D. Apply cool tap water compresses to the affected areas

What is the best action for a nurse to take to relieve a patient's pruritus? A. Rubbing the pruritic area B. Applying topical antiviral agents C. Applying a heating pad twice daily D. Applying cold, wet dressings to pruritic area

D. Applying cold, wet dressings to pruritic area Rationale Pruritus, or itching, can be caused by dry skin, almost any physical or chemical stimulus to the skin (such as drugs or insects), and any scaling skin disorder. Wet dressings may relieve pruritus. Topical antiviral agents, rubbing the area, and heating pads will not relieve the discomfort associated with this disorder. p. 424

The nurse is assessing a patient who had a face-lift procedure earlier in the day. The nurse notes that the skin in the surgical area is warm and pink and blanches with gentle pressure. What is the appropriate action by the nurse, based on this assessment finding? A. Apply moist heat to the area. B. Take the patient's temperature. C. Notify the health care provider immediately. D. Document the assessment finding as normal.

D. Document the assessment finding as normal. Rationale Postoperative assessment for the patient who has had a face-lift includes careful monitoring for adequate circulation. Warm, pink skin that blanches on pressure indicates that adequate circulation is present in the surgical area. Supportive, compressive dressings and ice packs (not moist heat) may be necessary early in the postoperative period. Moist heat should not be applied to the area. It is not necessary to take the patient's temperature. The health care provider does not need to be notified because this finding is normal. p. 425

Which surgical approach is used for coagulation of bleeding deep vessels to obtain hemostasis? A. Curettage B. Cryosurgery C. Electrodesiccation D. Electrocoagulation

D. Electrocoagulation Rationale Electrocoagulation involves the use of a dipolar electrode for coagulation and can affect deep blood vessels. Curettage is the removal and scooping away of the tissue using an instrument with a circular cutting edge attached to a handle. Cryosurgery is the use of subfreezing temperatures to destroy epidermal lesions. Electrodesiccation is useful for superficial vessels. STUDY TIP: Develop a realistic plan of study. Do not set rigid, unrealistic goals. p. 422

A patient who has redness around a hair follicle and necrotic debris in the ruptured material undergoes incision and drainage of painful nodules. The patient's temperature is 100° F. Which skin infection does the patient have? A. Furuncle B. Carbuncle C. Folliculitis D. Furunculosis

D. Furunculosis Rationale Furunculosis is characterized by a tender, erythematous area around a hair follicle with drainage of pus and a core of necrotic debris on rupture. Furunculosis is also marked by clinical manifestations such as malaise, regional adenopathy, and increased body temperature. This patient, who displays most of the symptoms, may have furunculosis. A patient with furuncle has lesions similar to the furunculosis lesion but will not exhibit malaise, regional adenopathy, or increased body temperature. Carbuncle is marked by the appearance of many pustules in an erythematous area, most commonly at the nape of the neck. A patient with folliculitis has small pustules at hair follicle openings and minimal erythema. Test-Taking Tip: Sometimes the reading of a question in the middle or toward the end of an exam will trigger your mind to come up with the answer or provide an important clue to an earlier question. p. 414

A patient who is obese and has a diagnosis of diabetes is at risk for a bacterial skin infection. For what infection should the nurse educate the patient to monitor? A. Cellulitis B. Impetigo C. Carbuncle D. Furunculosis

D. Furunculosis Rationale Patients who are obese and diabetic are at increased risk for furunculosis because of the likelihood of skin folds containing excess moisture. Therefore the nurse should warn the patient about the possibility of furunculosis. Cellulitis may be a secondary complication or a primary infection; it most often results from a break on the skin's surface. Impetigo is most common in patients with poor hygiene. Many factors are responsible for the development of carbuncle. p. 414

A patient informs the nurse that he or she has tingling and burning on the lower lip. The nurse assesses redness and a group of vesicles on the lower lip. Which infection should the nurse educate the patient? A. Impetigo B. Candidiasis C. Herpes zoster D. Herpes simplex virus

D. Herpes simplex virus Rationale Infection caused by herpes simplex virus is characterized by single or grouped vesicles on an erythematous base with a painful local reaction. Impetigo is a bacterial infection characterized by vesiculopustular lesions that develop a thick, honey-colored crust surrounded by erythema. Candidiasis is a fungal infection of the skin characterized by cheesy white plaques in the mouth and diffuse papular erythematous rash with pinpoint satellite lesions around the edges of the affected area. Herpes zoster, which is also called shingles, is characterized by linear distribution of vesicles along a dermatome. It is usually unilateral. p. 415

The nurse is caring for a patient in whom chickenpox is suspected. The patient has pustules and redness only on the left side of the face. Which skin condition may the patient have? A. Furuncle B. Candidiasis C. Verruca vulgaris D. Herpes zoster (shingles)

D. Herpes zoster (shingles) Rationale The patient may have herpes zoster (shingles), marked clinically by a linear distribution of vesicles or pustules along a dermatome on an erythematous base. The lesions resemble those of chickenpox. Herpes zoster appears unilaterally on the trunk, face, and lumbosacral areas. Furuncle is a bacterial infection characterized by a tender erythematous area around a hair follicle; it is most common on the face, back of the neck, axillae, breasts, and buttocks. Candidiasis is characterized by a diffuse papular erythematous rash with pinpoint satellite lesions around the edges of affected area. Patients with verruca vulgaris have circumscribed, hypertrophic, flesh-colored papules that are painful when compressed laterally. p. 415

A patient has itchy red papules that are circumscribed with vesicles. Which drug treatments should the nurse expect to be prescribed for this patient? Select all that apply. A. Cetirizine B. Mupirocin C. Fluorouracil D. Hydroxyzine E. Corticosteroids

D. Hydroxyzine E. Corticosteroids Rationale Itchy red papules circumscribed with vesicles and pruritus indicate allergic contact dermatitis. The patient should be treated with an antihistamine, such as hydroxyzine, for pruritus and an allergic condition. Corticosteroids are also used to control pruritus. Cetirizine is an antihistamine but it binds to peripheral histamine receptors and does not cause sedation or control pruritus. Mupirocin is a topical antibiotic used in the treatment of bacterial skin infections such as impetigo. Fluorouracil is a topical cytotoxic agent used in the treatment of premalignant and some malignant skin diseases. p. 421

The nurse is teaching the residents of an independent living facility about preventing skin infections and infestations. What should be included in the teaching? A. Use cool compresses if an infection occurs. B. Oral antibiotics will be needed for any skin changes. C. Antiviral agents will be needed to prevent outbreaks. D. Inspect skin for changes when bathing with mild soap.

D. Inspect skin for changes when bathing with mild soap. Rationale Individuals living in independent living facilities are usually older, which means their skin does not need cleaning with hot water and vigorous scrubbing or as often as a younger person. Mild soap (e.g., Ivory) should be used to avoid loss of protection from neutralization of the skin's surface. The skin should be inspected for changes while bathing. Cool compresses are used with ringworm or stings for the antiinflammatory effect. Oral antibiotics are used for Lyme disease from ticks. Antiviral agents are used for viral infections but not to prevent outbreaks. Test-Taking Tip: Make certain that the answer you select is reasonable and obtainable under ordinary circumstances and that the action can be carried out in the given situation. p. 409

The nurse is preparing to administer a topical corticosteroid preparation to a patient who has a severe skin inflammation. Which of these forms of topical corticosteroids is the most potent delivery system for this agent? A. Gel B. Spray C. Cream D. Ointment

D. Ointment Rationale The most potent delivery system for a topical corticosteroid is an ointment form. Gels, sprays, and creams are not as potent. Test-Taking Tip: Prepare for exams when and where you are most alert and able to concentrate. If you are most alert at night, study at night. If you are most alert at 2 am, study in the early morning hours. Study where you can focus your attention and avoid distractions. This may be in the library or in a quiet corner of your home. The key point is to keep on doing what is working for you. If you are distracted or falling asleep, you may want to change when and where you are studying. p. 421

The nurse is caring for four patients. Which patient will benefit from treatment with gabapentin for postherpetic neuralgia? A. Patient A: Group of painful vesicles on a red patch of skin; body temperature of 103 F. B: Patient B: Wart on the bottom surface of the foot that is growing inward and is painful when pressure is applied. C. Patient C: Presence of a circumscribed, large and growing, flesh-colored papule that is painful when the sides are pressed. D. Patient D: Linearly distributed vesicles and pustules and redness of the skin, associated with burning pain present only on the right side of the back.

D. Patient D: Linearly distributed vesicles and pustules and redness of the skin, associated with burning pain present only on the right side of the back. Rationale Unilateral linear distribution of vesicles and papules on an erythematous base, a burning sensation, and pain are the symptoms of herpes zoster (shingles). Patient D may experience postherpetic neuralgia, for which gabapentin is beneficial. Patient A has a group of vesicles on an erythematous base and a fever; these are clinical manifestations of herpes simplex virus (HSV) infection. This patient may benefit from treatment with such antiviral agents as acyclovir, famciclovir, and valacyclovir. Patient B has a wart on the bottom of his foot that is painful when pressure is applied. This is a plantar wart and requires treatment with topical immunotherapy, salicylic acid, and cryosurgery. Patient C has a circumscribed, hypertrophic, flesh-colored papule that is painful when compressed laterally. This is verruca vulgaris; treatment options include liquid nitrogen therapy, cantharidin, and salicylic acid. Test-Taking Tip: Assess each patient and distinguish between the symptoms to identify the infection in the patients. Then, apply the knowledge to answer the question. p. 415

The nurse is assessing four patients. Which patient should be advised to refrain from having sex? A. Patient A: Red, blistering scales between the toes. B: Patient B: Scaly, red, ringlike lesions with a defined margin. C. Patient C: The nails on two toes are brittle, thickened, and broken with yellow discoloration. D. Patient D: Red, edematous, painful vaginal wall marked with white patches.

D. Patient D: Red, edematous, painful vaginal wall marked with white patches. Rationale Patient D has a red, edematous, painful vaginal wall with white patches. These findings indicate that Patient D has candidiasis and should be advised to refrain from having sex. Patient A has interdigital scaling with erythema and blistering, suggesting tinea pedis. Patient B has a scaly, annular lesion with a well-defined margin, which is a clinical manifestation of tinea corporis. Patient C has several toenails that are brittle, thickened, broken, and yellowed, indicative of tinea unguium (onychomycosis). Test-Taking Tip: Each patient is displaying clinical manifestations of certain skin infections. Identify the infection affecting each patient and apply that knowledge to answer the question. p. 418

The nurse is caring for patients at a camp with bites and stings inflicted by insects or arachnids. Which patient may require the application of topical corticosteroids? A. Patient bitten by lice B. Patient stung by a bee C. Patient bitten by mites D. Patient bitten by bedbugs

D. Patient bitten by bedbugs Rationale Bedbug bites are treated with the use of antihistamines and topical corticosteroids. Lice bites cause minute, red noninflamed points that are flush with the skin. Pyrethrins and spinosad are used in the treatment of pediculosis (lice infestation). People who are stung by bees or wasps may experience intense burning and local pain at the site of the bite, along with swelling and itching. The sting may be treated with cool compresses and antihistamines as necessary. The patient bitten by mites may have scabies, often marked by severe itching and crusting between the fingers. Scabies is treated with a 5% topical lotion of permethrin. p. 416

The nurse is caring for a patient who has lost excessive hair from the scalp and some other parts of the body. The patient is also unable to sweat. Which therapy does the nurse suspect that the patient has been undergoing? A. Drug therapy B. Phototherapy C. Laser technology D. Radiation therapy

D. Radiation therapy Rationale Radiation therapy can cause alopecia (permanent hair loss) of the irradiated areas and temporary impairment of the sweat glands. Drug therapy includes the use of various drugs such as antibiotics, corticosteroids, antihistamines, topical fluorouracil, and immunomodulators; drug therapy is not generally associated with hair loss and sweat gland impairment. A patient undergoing phototherapy will experience erythema, sunburn, and persistent pruritus. Laser light does not accumulate in the body cells and cannot produce cellular changes or damage. Test-Taking Tip: Identify option components as correct or incorrect. This may help you identify a wrong answer. p. 420

During the assessment of a patient, the nurse notes an area of irregularly round verrucous papules with well-defined shapes. The patient states that they have become darker over the past few months and are often itchy and irritating. The nurse recognizes this finding as what? A. Lentigo B. Psoriasis C. Acne vulgaris D. Seborrheic keratosis

D. Seborrheic keratosis Rationale Clinical manifestations of seborrheic keratosis include irregularly round or oval, often verrucous papules or plaques with well-defined shape and the appearance of being stuck on. The lesions increase in pigmentation with time and are usually multiple and possibly itchy Clinical manifestations of lentigo include hyperpigmented, brown to black macule or patch (flat lesion) over sun-exposed areas. Clinical manifestations of psoriasis include sharply demarcated silvery scaling plaques on reddish colored skin commonly on the scalp, elbows, knees, palms, soles, and fingernails. Acne vulgaris is manifested by noninflammatory lesions, including open comedones (blackheads) and closed comedones (whiteheads), and inflammatory lesions, including papules and pustules. p. 419

Which localized finding would the nurse expect in the patient with diabetes recently diagnosed with cellulitis? A. Chills B. Fever C. Fatigue D. Swelling

D. Swelling Rationale Signs of cellulitis are local; swelling is a localized finding. Chills, fever, and fatigue are systemic (generalized). Test-Taking Tip: Be aware that information from previously asked questions may help you respond to other examination questions. p. 414

The nurse assesses a scaly, ringlike rash with a clear center on a patient's leg. Regarding which skin infection should the nurse educate the patient? A. Cellulitis B. Tinea cruris C. Plantar warts D. Tinea corporis

D. Tinea corporis Rationale Tinea corporis is marked by a scaly annular rash with a well-defined margin that is erythematous. Cellulitis is a bacterial infection characterized by a hot, tender, erythematous, and edematous area with a diffuse border. Tinea cruris is characterized by well-defined scaly plaques in the groin. Plantar warts, caused by a viral infection, are found on the bottom of the foot and are painful when pressure is applied. p. 418

A patient with skin lesions is advised to have a patch test. The nurse applies the patch on the patient's skin and instructs the patient to come back after two days for further treatment. What is the likely reason the patch test was performed? A. To determine the presence of melanoma in the patient. B. To determine the prognosis of skin cancer in the patient. C. To determine the presence of skin infections in the patient. D. To determine the causative agents of allergies in the patient.

D. To determine the causative agents of allergies in the patient. Rationale The patch test is done to identify the cause of allergies in patients. It is performed by applying the allergen on the skin of the patient and evaluating the skin after 48 hours. The presence of erythema, papules, and vesicles indicate an allergic reaction. A patch test is not useful to determine the presence or prognosis of skin cancer. The prognosis of skin cancer can be determined through Breslow's and Clark's measurements. The presence of skin infections can be determined through microscopic studies. The presence of melanoma can be determined through biopsies. p. 409

A patient with a history of persistent macular eruption notices the gradual appearance of hard, reddened plaques on his or her trunk that are similar in appearance to psoriasis lesions. Which treatment does the nurse suspect will be beneficial in this patient? A. Chemical peels B. Surgical excision C. Electrodesiccation D. Topical nitrogen mustard

D. Topical nitrogen mustard Rationale Mycosis fungoides is a form of cutaneous T-cell lymphoma in which lymphocytes become malignant, affecting the skin. It is characterized by indurated erythematous plaques on the trunk that are similar in appearance to psoriasis lesions. Therefore topical nitrogen mustard, a chemotherapy agent, will be beneficial. Chemical peeling is useful against actinic keratosis. Surgical excision is a treatment option for both basal cell carcinoma and squamous cell carcinoma. Squamous cell carcinoma, a malignant tumor of squamous cells of the epidermis, requires treatment with electrodesiccation, in which the skin growths are scraped or burned away. p. 412

A patient has sensitive skin and breaks out with rashes when using ordinary soap. What advice should the nurse give the patient regarding hygienic skin measures? A. Rub the skin vigorously. B. Avoid antibacterial soaps. C. Use hot water to take a bath. D. Use mild, moisturizing soaps.

D. Use mild, moisturizing soaps. Rationale People with sensitive skin should use mild, moisturizing soaps and lipid-free cleansers. The patient should avoid hot water and vigorous scrubbing, because they may cause local skin irritation and inflammation. Antibacterial soaps without sulfites should be used on the areas with skin piercings where jewelry has been inserted. p. 409

A nurse is assessing a patient who reports extreme fatigue and muscle pain. The patient reports spending very little time outdoors. The skeletal muscles of the patient are very weak. Which deficiency is most likely to cause such symptoms? A. Vitamin A deficiency B. Vitamin B deficiency C. Vitamin C deficiency D. Vitamin D deficiency

D. Vitamin D deficiency Rationale Muscle pain and weakness are signs of Vitamin D deficiency. Vitamin D is produced naturally by cutaneous photosynthesis due to exposure to UVB light. The patient is most likely to have Vitamin D deficiency because of restricted sun exposure. Vitamin C deficiency causes scurvy, which is characterized by purpura, bleeding gums and petechiae. Deficiency of vitamin A causes dryness of the conjunctiva and poor wound healing. Deficiency of Vitamin B causes dermatologic symptoms such as erythema, bullae, and seborrhea-like lesions. p. 409

What action should the nurse take to help a patient with pruritus? A. Keep the area as dry as possible. B. Administer fexofenadine to control itching. C. Allow the patient to gently rub the itching area for quick relief. D. Wet cotton sheets with warm water and place them over the itching area.

D. Wet cotton sheets with warm water and place them over the itching area. Rationale Wet dressings relieve pruritus (itching). Thin cotton sheets should be placed in warm water, wrung out, and placed over the pruritic area for 10 to 15 minutes. The pruritic area should be kept moistened with lubricant or medication because dryness of the skin increases the itch sensation. Although systemic antihistamines provide symptomatic relief, sedating antihistamines such as hydroxyzine and diphenhydramine are used for pruritus because nonsedating antihistamines such as fexofenadine are not effective. Anything that causes vasodilation, such as heat or rubbing, should be avoided because it makes the itching worse. p. 424

The nurse reviews lab values for a male patient with herpes zoster. With which result should the nurse be most concerned? A. Calcium: 9.0 mg/dL B. Hemoglobin: 14 g/dL C. Platelets: 150,000/mm 3 D. White blood cell count: 1000/mm 3

D. White blood cell count: 1000/mm 3 Rationale Herpes zoster may occur as reactivation of the varicella virus, which is dormant in the ganglion after a primary case of chickenpox. Reactivation is seen in immunocompromised patients. The nurse would be concerned about the patient's immune status, and therefore a check of the white blood cell count would be warranted. A normal white blood cell count is 4000 to 10,000 mm 3. The other answer options all contain lab values within normal limits: platelets 150,000 to 350,000/mm 3, hemoglobin 13 to 18 g/dL in males and 12 to 16 g/dL in females, and calcium 8.5 to 10.5 mg/dL. p. 415

The nurse would assess a patient admitted with cellulitis for what localized manifestation? A. Pain B. Fever C. Chills D. Malaise

A. Pain

The nurse is providing preoperative teaching for the patient having a face-lift (rhytidectomy) surgery. Which patient response indicates the patient understands the teaching? A. "I am afraid of the pain afterwards, while it is healing." B. "I can't wait to have my forehead and lip wrinkles eliminated." C. "I have some time off work so I will not look so bad when I go back." D. "Now I can be excited to go to my 50th high school reunion this week."

C. "I have some time off work so I will not look so bad when I go back." Rationale A rhytidectomy or face-lift surgery will not have immediate results and will take time to heal, so taking time off from work will allow more healing to be accomplished before returning to work. There is not much pain with most cosmetic surgeries. A rhytidectomy will not eliminate forehead lines and vertical lip wrinkles. Test-Taking Tip: Watch for grammatical inconsistencies. If one or more of the options is not grammatically consistent with the stem, the alert test taker can identify it as a probable incorrect option. When the stem is in the form of an incomplete sentence, each option should complete the sentence in a grammatically correct way. p. 425

A patient is diagnosed with herpes zoster and is at risk for developing postherpetic neuralgia. What action should the nurse take to prevent postherpetic neuralgia in the patient? A. Administer mild sedatives at night. B. Apply wet compresses to the affected area. C. Administer acyclovir as prescribed within 72 hours. D. Apply silver sulfadiazine on the ruptured vesicles.

C. Administer acyclovir as prescribed within 72 hours.

The nurse assesses circumscribed, hypertrophic, flesh-colored papules on a patient's knee. The patient states they are recurring even after removal. How should the nurse document these findings? A. Plantar warts B. Herpes zoster C. Verruca vulgaris D. Herpes simplex virus type 1

C. Verruca vulgaris Rationale Verruca vulgaris is characterized by circumscribed, hypertrophic, flesh-colored papules that are limited to the epidermis. Plantar warts are found on the bottom surface of the foot; they may grow inward as a result of the pressure of walking and standing. Herpes zoster is characterized by linear distribution along a dermatome of grouped vesicles and pustules on an erythematous base. Herpes simplex virus type 1's clinical manifestations are single or grouped vesicles on an erythematous base occurring with systematic symptoms of fever and malaise. p. 415

A patient has developed a skin infection with an associated fever of 101.6° F, white blood cell count of 28,000, headache, and malaise. With what skin infection does the nurse determine these clinical manifestations correlate? A. Impetigo B. Carbuncle C. Folliculitis D. Erysipelas

D. Erysipelas


Set pelajaran terkait

Chapter 1: Preview of Cell Biology

View Set

Mindfulness, Compassion, Self-Compassion EXAM 2

View Set

Chapter 13: DEVELOPING THE IMPLEMENTATION PLAN:

View Set

S3: U5: Pain: Practice Questions

View Set

Series 7 Chapter 14: Economics & Analysis

View Set

Petroleum Engineering Flashcards (Gas Liquification - Seismic)

View Set